If Q is more popular than L, then each of the following must be true of the ranking EXCEPT:

ishadoshi on August 9, 2019

Understanding questions

This is in response to the message board started by Savannah on December 21 at 7:56 pm. I couldn't respond there directly for some reason so I'm typing it out here: by drilling do you mean we should do the same logic game over and over again until we get it? if yes, then would that really help much? since after seeing the video explanation of the game, we know how to set it up and therefore it will just be like replicating what we saw in the video - thereby not really being an accurate test of whether we've really understood the question?

Replies
Create a free account to read and take part in forum discussions.

Already have an account? log in

ChrisH85 on September 15, 2019

How do I answer this question? Can you help me rewrite out what the rule would look like here?

shunhe on January 7, 2020

Hi @ishadoshi,

I would say that it's helpful to revisit logic games and do them without video guidance. You don't necessarily need to do it twice in a row in the same hour - do a logic game, work through some others, and a few days or weeks later return to a logic game you worked on in the past and do it without any of the outside help. That's what I think will help a lot. There's a huge difference between being able to follow along with an explanation of a logic game and being able to do one yourself. Hope this helps!

shunhe on January 7, 2020

Hi @ChrisH85,

Thanks for your questions. I'm not sure what you mean by "rewrite out what the rule would look like here," but happy to walk through the question, and if I don't answer your second question, please ask again and let me know what rule you're talking about!

So given the information in the question, which tells us Q is more popular than L, we can amend our diagram to write:


P
/
H - J - Q - S
/
L - V

And this is a must be true EXCEPT. One of these answer choices, then, could be false.

(A) is incorrect because it must be true that (H) is first, since everything else comes after it.
(B) is the correct answer. The first three letters have to be H, J, and Q. But the fourth letter could be any one of P, S, and V, and so we know that (B) could be false; L doesn't have to be fourth.
(C) is incorrect because V can't be fourth. There are four letters that come before it.
(D) is incorrect because J can't be third. There's only one letter in front of it.
(E) is incorrect because Q has to be third. There's only two letters in front of it.

Hope this helps!